\collectexercises{banque} \begin{exercise}[subtitle={Éléments remarquables du logarithme}, step={1}, topics={Logarithme}] \begin{enumerate} \item Tracer l'allure de la courbe représentative du logarithme. \item Repérer les éléments remarquables de cette représentation graphique. \item Tracer le tableau de signe de $\ln$. \item Tracer le tableau de variation de $\ln$. \end{enumerate} \end{exercise} \begin{exercise}[subtitle={Dériver les fonctions}, step={1}, topics={Logarithme}] Dériver les fonctions suivantes puis mettre sous une forme pratique pour l'étude de signe. \begin{multicols}{3} \begin{enumerate} \item $f(x) = x-2-\ln(x)$ \item $f(x) = 2x^2 - 2x + 4\ln(x)$ \item $f(x) = x\ln(x)$ \item $f(x) = (x+1)\ln(x)$ \item $f(x) = (\ln(x) + 1)^2$ \item(*) $f(x) = \frac{1 + \ln(x)}{x}$ \end{enumerate} \end{multicols} \end{exercise} \begin{exercise}[subtitle={Dériver les fonctions - Bis}, step={1}, topics={Logarithme}] Dériver les fonctions suivantes puis mettre sous une forme pratique pour l'étude de signe. \begin{multicols}{3} \begin{enumerate} \item $f(x) = 2x - \ln(x) + 2$ \item $f(x) = x^3 - 4\ln(x)$ \item $f(x) = e^{3x} + 2 $ \item $f(x) = (2x - 2)\ln(x)$ \item(*) $f(x) = (\ln(x) + 1)(3x+2)$ \item(*) $f(x) = \frac{\ln(x)}{x}$ \end{enumerate} \end{multicols} \end{exercise} \begin{exercise}[subtitle={Étude de fonction}, step={2}, topics={Logarithme}] On considère la fonction $f$ définie sur $\intFF{1}{11}$ par \[ f(x) = -0.5x^2 + 2x + 15\ln(x) \] \begin{enumerate} \item Démontrer que la dérivée de $f$ est \[ f'(x) = \frac{-x^2 + 2x + 15}{x} \] \item Étudier le signe de $f'$ et en déduire les variations de $f$. \item Montrer que l'équation $f(x) = 0$ admet une unique solution, $\alpha$, sur $\intFF{1}{11}$. \item Donner une valeur approchée de $\alpha$. \item En déduire le tableau de signe de $f$. \end{enumerate} \end{exercise} \begin{exercise}[subtitle={Étude de fonction}, step={2}, topics={Logarithme}] On considère la fonction $f$ définie sur $\intFO{0}{+\infty}$ par \[ f(x) = \frac{1 + \ln(x)}{x} \] \begin{enumerate} \item Démontrer que la dérivée de $f$ est \[ f'(x) = \frac{-\ln(x)}{x^2} \] \item Étudier le signe de $f'$ et en déduire les variations de $f$. \item Déterminer le minimum de la fonction $f$. \item En déduire le tableau de signe de $f$. \end{enumerate} \end{exercise} \begin{exercise}[subtitle={Recherche par dichotomie}, step={2}, topics={Logarithme}] On considère la fonction $f$ définie sur $\intFF{1}{5}$ par \[ f(x) = 3x -10 + 4\ln(x) \] \begin{enumerate} \item \begin{enumerate} \item Démontrer que la dérivée de $f$ est \[ f'(x) = \frac{3x + 4}{x} \] \item Étudier le signe de $f'$ et en déduire les variations de $f$. \item Montrer que l'équation $f(x) = 0$ admet une unique solution, $\alpha$, sur $\intFF{1}{5}$. \end{enumerate} \item On souhaite trouver un encadrement de $\alpha$ par la méthode de dichotomie. Pour cela, on propose l'algorithme suivant: \begin{center} \begin{minipage}{0.5\textwidth} \begin{algorithm}[H] \SetAlgoLined $a \leftarrow 1$ \; $b \leftarrow 5$ \; \Tq{$b-a \geq 0.01$}{ $m \leftarrow \dfrac{b+a}{2}$ \; \eSi{f(m) < 0}{ $a \leftarrow m$\; }{ $b \leftarrow m$\; } } \Retour{$a, b$} \end{algorithm} \end{minipage} \end{center} \begin{enumerate} \item En vous aidant du tableau ci-dessous (vous pouvez ajouter des lignes si nécessaire) exécuter l'algorithme pour trouver un encadrement d'amplitude 0.01 de $\alpha$. \begin{center} \begin{tabular}{|*{5}{p{2cm}|}} \hline $a$ & $b$ & $(b-a) \geq 0.01$ & $m$ & $f(m) < 0$ \\ \hline & & & & \\ \hline & & & & \\ \hline & & & & \\ \hline \end{tabular} \end{center} \item Expliquer le fonctionnement de cet algorithme en quelques phrases. \end{enumerate} \end{enumerate} \end{exercise} \begin{exercise}[subtitle={Primitives}, step={3}, topics={Logarithme}] Calculer les primitives des fonctions suivantes \begin{multicols}{2} \begin{enumerate} \item $f(x) = \dfrac{1}{x} + x$ \item $g(x) = -\dfrac{4}{x}$ \item $h(x) = 5x + \dfrac{10}{x}$ \item $i(t) = 2t + \dfrac{4}{t} - 2\dfrac{1}{t^2}$ \end{enumerate} \end{multicols} \end{exercise} \begin{exercise}[subtitle={Coût de fabrication}, step={3}, topics={Logarithme}] Un usine fabrique entre \np{1000} et \np{7000} objets pas semaines. Un étude des moyens de productions a permis de modéliser les coûts de production par la fonction $C$ définie sur $\intFF{1}{7}$ par \[ c(x) = 1,5x^2 - 9x + 24 + \dfrac{48}{x} \] où $x$ représente la production hebdomadaire en milliers d'objets. \begin{enumerate} \item Étude des variations des coûts. \begin{enumerate} \item Démontrer que la dérivée de $c$ est \[ c'(x) = \frac{3(x-4)(x^2+x+4)}{x^2} \] \item Étudier le signe de $c'$ et en déduire les variations de $c$ \item Pour quelle production les coûts de productions sont-ils minimal? \end{enumerate} \item Étude des coûts de productions moyen. \begin{enumerate} \item Démontrer qu'une primitive de $c$ est \[ C(x) = 0.5x^3 - 4,5x^2 + 24x + 1 + 48\ln(x) \] \item Calculer la valeur moyenne de $c$ sur $\intFF{1}{7}$. \item Interpréter le résultat précédent. \end{enumerate} \end{enumerate} \end{exercise} \begin{exercise}[subtitle={Coût de fabrication}, step={4}, topics={Logarithme}] % Polynésie Juin 2019 Ex 4 \begin{center}\textbf{Les deux parties de cet exercice sont indépendantes}\end{center} \textbf{Partie A} \medskip Une entreprise produit chaque année entre $100$ et $900$ pneus pour tracteurs. On considère la fonction $f$ définie sur l'intervalle [1~;~9] par \[f(x ) = 0,5 x^2 - 7x + 14 + 6\ln (x).\] On admet que la fonction $f$ modélise le coût moyen annuel de fabrication d'un pneu, exprimé en centaines d'euros, pour $x$ centaines de pneus produits. \medskip \begin{enumerate} \item La fonction $f$ est dérivable sur l'intervalle [1~;~9] et on note $f'$ sa fonction dérivée. Démontrer que pour tout réel $x$ de l'intervalle [1~;~9] on a : $f'(x)= \dfrac{x^2 -7 x + 6}{x}$. \item \begin{enumerate} \item Justifier les variations suivantes de la fonction $f$ sur l'intervalle [1~;~9] : \begin{center} \begin{tikzpicture}[baseline=(a.north)] \tkzTabInit[lgt=3,espcl=3]{$x$/1,Variations de\\ $f(x)$/2}{1, 6, 9} \tkzTabVar{+/ , -/ , +/ } \end{tikzpicture} \end{center} \item Justifier que, sur l'intervalle [1~;~9], l'équation $f(x) = 5$ admet une unique solution $\alpha$. \item Donner un encadrement au centième près de $\alpha$. \item On considère l'algorithme ci-dessous: \begin{center} \begin{tabularx}{0.5\linewidth}{|X|}\hline $X \gets 1$\\ $Y \gets 7,5$\\ Tant que $Y > 5$\\ \hspace{12mm}$X \gets X + 0,01$\\ \hspace{12mm}$Y \gets 0,5X^2 - 7X + 14 + 6*\ln (X)$\\ Fin Tantque\\ \hline \end{tabularx} \end{center} À la fin de l'exécution de l'algorithme, quelle valeur numérique contient la variable $X$? \end{enumerate} \item Pour quelle quantité de pneus, le coût moyen annuel de fabrication d'un pneu est-il minimal ? À combien s'élève-t-il ? \end{enumerate} \bigskip \textbf{Partie B} \medskip Cette même entreprise envisage la fabrication de semoirs (gros matériel agricole). On admet que la fonction $g$ définie sur l'intervalle [0~;~100] par \[g (x) = 2x -1 + \text{e}^{0,05x}\] modélise le coût de fabrication, exprimé en centaines d'euros, de $x$ semoirs. \medskip \begin{enumerate} \item Donner une primitive $G$ de la fonction $g$ sur l'intervalle [0~;~100]. \item Calculer la valeur moyenne de la fonction $g$ sur l'intervalle [0~;~100]. \item Interpréter ce résultat dans le contexte de l'exercice. \end{enumerate} \end{exercise} \begin{solution} \textbf{Partie A} \medskip Une entreprise produit chaque année entre $100$ et $900$ pneus pour tracteurs. On considère la fonction $f$ définie sur l'intervalle $\left [1~;~9\strut\right ]$ par $f(x ) = 0,5 x^2 - 7x + 14 + 6\ln (x)$. On admet que la fonction $f$ modélise le coût moyen annuel de fabrication d'un pneu, exprimé en centaines d'euros, pour $x$ centaines de pneus produits. \medskip \begin{enumerate} \item% La fonction $f$ est dérivable sur l'intervalle $\left [1~;~9\strut\right ]$ et on note $f'$ sa fonction dérivée. Pour tout réel $x$ de l'intervalle $\left [1~;~9\strut\right ]$ on a $f(x ) = 0,5 x^2 - 7x + 14 + 6\ln (x)$ donc\\ $f'(x)= 0,5\times 2x - 7 + 6\times \dfrac{1}{x} = x-7-\dfrac{6}{x} = \dfrac{x^2-7x+6}{x}$. %Démontrer que pour tout réel $x$ de l'intervalle $\left [1~;~9\strut\right ]$ on a : $f'(x)= \dfrac{x^2 -7 x + 6}{x}$. \item \begin{enumerate} \item On va justifier les variations suivantes de la fonction $f$ sur l'intervalle [1~;~9] : \begin{center} {\renewcommand{\arraystretch}{1.1} \psset{nodesep=3pt,arrowsize=2pt 3} % paramètres \def\esp{\hspace*{1.5cm}}% pour modifier la largeur du tableau \def\hauteur{0pt}% mettre au moins 20pt pour augmenter la hauteur $\begin{array}{|c| *4{c} c|} \hline x & 1 & \esp & 6 & \esp & 9 \\ \hline & \Rnode{max1}{\phantom{0}} & & & & \Rnode{max2}{\phantom{0}} \\ \text{Variations de } f & & & & & \rule{0pt}{\hauteur} \\ & & & \Rnode{min}{\phantom{0}} & & \rule{0pt}{\hauteur} \ncline{->}{max1}{min} \ncline{->}{min}{max2} %\rput*(-3.7,0.65){\Rnode{zero}{\blue 0}} %\rput(-3.7,1.7){\Rnode{alpha}{\blue \alpha}} %\ncline[linestyle=dotted, linecolor=blue]{alpha}{zero} %\rput*(-1.3,0.65){\Rnode{zero2}{\red 0}} %\rput(-1.3,1.7){\Rnode{beta}{\red \beta}} %\ncline[linestyle=dotted, linecolor=red]{beta}{zero2} \\ \hline \end{array}$ } \end{center} Sur $\left [1~;~9\strut\right ]$, $f'(x)$ est du signe de $x^2-7x+6$. $\Delta = 7^2 - 4\times 1\times 6 = 49-24=25=5^2$ donc le trinôme $x^2-7x+6$ admet deux racines:\\[3pt] $x'=\dfrac{-b-\sqrt{\Delta}}{2a} = \dfrac{7-5}{2}=1$ et $x''=\dfrac{-b+\sqrt{\Delta}}{2a}=\dfrac{7+5}{2}=6$. \smallskip On en déduit le signe du trinôme (positif à l'extérieur des racines) donc de $f'(x)$: \begin{center} \renewcommand{\arraystretch}{1.5} \def\esp{\hspace*{1.2cm}} $\begin{array}{|c | *{5}{c} |} \hline x & 1 & \esp & 6 & \esp & 9 \\ \hline f'(x) & & \pmb{-} & \vline\hspace{-2.7pt}{0} & \pmb{+} & \\ \hline \end{array}$ \end{center} Cela justifie les variations de $f$. \item% Justifier que, sur l'intervalle [1~;~9], l'équation $f(x) = 5$ admet une unique solution $\alpha$. On complète le tableau de variations de $f$: $f(1)=7,5$, $f(6)\approx 0,75$ et $f(9)\approx 4,7$ \begin{center} {\renewcommand{\arraystretch}{1.3} \psset{nodesep=3pt,arrowsize=2pt 3} % paramètres \def\esp{\hspace*{1.5cm}}% pour modifier la largeur du tableau \def\hauteur{0pt}% mettre au moins 20pt pour augmenter la hauteur $\begin{array}{|c| *4{c} c|} \hline x & 1 & \esp & 6 & \esp & 9 \\ \hline & \Rnode{max1}{7,5} & & & & \Rnode{max2}{\approx 4,7} \\ \text{Variations de } f & & & & & \rule{0pt}{\hauteur} \\ & & & \Rnode{min}{\approx 0,75} & & \rule{0pt}{\hauteur} \ncline{->}{max1}{min} \ncline{->}{min}{max2} \rput*(-4.5,0.65){\Rnode{zero}{\blue 5}} \rput(-4.5,1.7){\Rnode{alpha}{\blue \alpha}} \ncline[linestyle=dotted, linecolor=blue]{alpha}{zero} \\ \hline \end{array}$ } \end{center} On en déduit que sur $\left [1~;~9\strut\right ]$, l'équation $f(x)=5$ admet une solution unique $\alpha$. \item %Donner un encadrement au centième près de $\alpha$. $\left. \begin{array}{l} f(2)\approx 6,2 > 5\\ f(3)\approx 4,1 < 5 \end{array} \right\rbrace \Rightarrow \alpha \in \left [ 2~;\,3\strut\right ]$ \hfill $\left. \begin{array}{l} f(2,5)\approx 5,1 > 5\\ f(2,6)\approx 4,9 < 5 \end{array} \right\rbrace \Rightarrow \alpha \in \left [ 2,5~;\, 2,6 \strut\right ]$ $\left. \begin{array}{l} f(2,55)\approx 5,018 > 5\\ f(2,56)\approx 4,997 < 5 \end{array} \right\rbrace \Rightarrow \alpha \in \left [ 2,55~;\, 2,56 \strut\right ]$ \item On considère l'algorithme ci-dessous: \begin{center} \begin{tabularx}{0.5\linewidth}{|X|}\hline $X \gets 1$\\ $Y \gets 7,5$\\ Tant que $Y > 5$\\ \hspace{12mm}$X \gets X + 0,01$\\ \hspace{12mm}$Y \gets 0,5X^2 - 7X + 14 + 6*\ln (X)$\\ Fin Tantque\\ \hline \end{tabularx} \end{center} À la fin de l'exécution de l'algorithme, la variable $X$ contient la valeur $2,56$, première valeur au centième pour laquelle $Y>5$. \end{enumerate} \item Le coût moyen annuel de fabrication d'un pneu est minimal quand la fonction $f$ atteint son minimum c'est-à-dire pour $x=6$; c'est donc pour la fabrication de 600 pneus que le coût moyen annuel de fabrication d'un pneu est minimal. Ce coût est, en euro, de $f(6)\times 100 \approx 75$. %À combien s'élève-t-il ? \end{enumerate} \bigskip \textbf{Partie B} \medskip Cette même entreprise envisage la fabrication de semoirs (gros matériel agricole). On admet que la fonction $g$ définie sur l'intervalle $\left [0~;~100\strut\right ]$ par $g (x) = 2x -1 + \e^{0,05x}$ modélise le coût de fabrication, exprimé en centaines d'euros, de $x$ semoirs. \medskip \begin{enumerate} \item %Donner une primitive $G$ de la fonction $g$ sur l'intervalle $\left [0~;~100\strut\right ]$. Sur l'intervalle $\left [0~;~100\strut\right ]$, la fonction $g$ a pour primitive la fonction $G$ définie par\\ $G(x)=x^2 - x + \dfrac{\e^{0,05x}}{0,05} = x^2-x +20\e^{0,05x}$. \item La valeur moyenne $m$ de la fonction $g$ sur l'intervalle $\left [0~;~100\strut\right ]$ est: $m=\dfrac{1}{100-0} \ds\int_{0}^{100} g(x) \d x = \dfrac{1}{100} \left [G(100) - G(0) \strut\right ] =\dfrac{1}{100}\left [ \left ( \np{9900} + 20\e^{5}\right ) - \left ( 20\right ) \right ] = \np{9880} +20\e^{5}\\[3pt] \phantom{m} \approx \np{128,46}$ \item %Interpréter ce résultat dans le contexte de l'exercice. Le coût moyen d'un semoir est donc, en euro, $128,46 \times 100 = \np{12846}$. \end{enumerate} \end{solution} \begin{exercise}[subtitle={Étude de fonction}, step={4}, topics={Logarithme}] % Métropole Septembre 2019 Ex 3 La courbe $\mathcal{C}_f$ ci-dessous est la courbe représentative d'une fonction $f$ définie et deux fois dérivable sur l'intervalle [1,1~;~8]. \begin{center} \begin{tikzpicture}[yscale=0.5, xscale=1.5, domain=1:8] \tkzInit[xmin=0,xmax=9,xstep=1, ymin=0,ymax=12,ystep=1] \tkzGrid \tkzAxeXY[up space=0.5,right space=.5] \clip (1.1, 0) rectangle (9,12); \tkzFct[line width=1pt]{(2.*x-1.)/log(x)} \end{tikzpicture} \end{center} \textbf{Les parties A et B peuvent être traitées de manière indépendante.} \bigskip \textbf{Partie A : étude graphique} \medskip \begin{enumerate} \item Donner une valeur approchée du minimum de la fonction $f$ sur l'intervalle [1,1~;~8] \item Quel est le signe de $f'(5)$ ? Justifier. \item Encadrer l'intégrale $\ds \int_2^4 f(x)\; dx$ par deux entiers consécutifs. \item La fonction $f$ est-elle convexe sur [1,1~;~3] ? Justifier. \end{enumerate} \bigskip \textbf{Partie B : étude analytique} \medskip On admet que $f$ est la fonction définie sur l'intervalle [1,1~;~8] par \[f(x) = \dfrac{2x - 1}{\ln (x)}.\] \smallskip \begin{enumerate} \item Montrer que, pour tout réel $x$ de l'intervalle [1,1~;~8], on a : \[f'(x) = \dfrac{2\ln (x) - 2 + \frac{1}{x}}{(\ln (x))^2}\] \item Soit $h$ la fonction définie sur [1,1~;~8] par : $h(x) = 2\ln (x) - 2 + \frac{1}{x}$. \begin{enumerate} \item Soit $h'$ la fonction dérivée de $h$ sur l'intervalle [1,1; 8]. Montrer que, pour tout réel $x$ de l'intervalle [1,1~;~8], \[h'(x) = \dfrac{2x - 1}{x^2}.\] \item En déduire les variations de la fonction $h$ sur l'intervalle [1,1~;~8]. \item Montrer que l'équation $h(x) = 0$ admet une unique solution $\alpha$ sur l'intervalle [1,1~;~8]. Donner un encadrement de $\alpha$ par deux entiers consécutifs. \end{enumerate} \item Déduire des résultats précédents le signe de $h(x)$ sur l'intervalle [1,1~;~8]. \item À l'aide des questions précédentes, donner les variations de [ sur [1,1~;~8]. \end{enumerate} \end{exercise} \begin{exercise}[subtitle={Loi de Benfort}, step={4}, topics={Logarithme}] % Métropole Juin 2017 Ex 4 Dans cet exercice, on considère le premier chiffre des entiers naturels non nuls, en écriture décimale. Par exemple, le premier chiffre de \np{2017} est 2 et le premier chiffre de 95 est 9. Dans certaines circonstances, le premier chiffre d'un nombre aléatoire non nul peut être modélisé par une variable aléatoire $X$ telle que pour tout entier $c$ compris entre 1 et 9, \[P(X = c) = \dfrac{\ln (c + 1) - \ln (c)}{\ln(10)}.\] Cette loi est appelée loi de Benford. \medskip \begin{enumerate} \item Que vaut $P(X = 1)$ ? \item On souhaite examiner si la loi de Benford est un modèle valide dans deux cas particuliers. \begin{enumerate} \item \textbf{Premier cas} Un fichier statistique de l'INSEE indique la population des communes en France au 1\ier{} janvier 2016 (champ: France métropolitaine et départements d'outre-mer de la Guadeloupe, de la Guyane, de la Martinique et de la Réunion). À partir de ce fichier, on constate qu'il y a \np{36677} communes habitées. Parmi elles, il y a \np{11094} communes dont la population est un nombre qui commence par le chiffre 1. Cette observation vous semble-t-elle compatible avec l'affirmation : \og{}le premier chiffre de la population des communes en France au 1 er janvier 2016 suit la loi de Benford \fg{} ? \item \textbf{Deuxième cas} Pour chaque candidat au baccalauréat de la session 2017, on considère sa taille en centimètres. On désigne par $X$ la variable aléatoire égale au premier chiffre de la taille en centimètres d'un candidat pris au hasard. La loi de Benford vous semble-t-elle une loi adaptée pour $X$ ? \end{enumerate} \end{enumerate} \end{exercise} \collectexercisesstop{banque}